The z- score for a mark of 56 in a test with a mean of 60 and standard deviation of 8 is​

Answers

Answer 1

Answer:

z = - .5

Step-by-step explanation:

z = (x - μ)/ σ

z = 56 - 60/8 = -.5


Related Questions

A farmer builds a fence to enclose a rectangular pasture. He uses 155 feet of fence. Find the total area of the pasture if it is 45.5 feet long. The length is 6 and the height is 5 what is the width?

Answers

Answer: [tex]1456\ ft^2[/tex]

Step-by-step explanation:

Given

Length of whole fence is 155 feet

If the length of rectangle is 45.5 ft

Suppose width is w

Length of whole fence is perimeter which is given by

[tex]\Rightarrow 155=2(45.5+w)\\\Rightarrow 77.5=45.5+w\\\Rightarrow w=32\ ft[/tex]

Area of the rectangle is given by the product of length and width

[tex]\Rightarrow A=lw\\\Rightarrow A=45.5\times 32\\\Rightarrow A=1456\ ft^2[/tex]

Thus, total area of pasture is [tex]1456\ ft^2[/tex]

A parabola opens upward. The parabola goes through the point (3,-1),
and the vertex is at (2,-2).

Find the value of A for the parabola. Show your work. Use Part 1 and 2 to write the equation of the parabola.

Answers

Answer:

a=1

Step-by-step explanation:

Hopefully this helps :)

The equation of the parabola is: y = (x - 2)² - 2. Finding the value of A

The vertex of the parabola is at (2,-2). Since the parabola opens upward, the equation of the parabola will be of the form:

y = A(x - 2)² - 2

We can plug the point (3,-1) into this equation to find the value of A.

-1 = A(3 - 2)² - 2

Simplifying the right side of the equation, we get:

-1 = A - 2

Adding 2 to both sides of the equation, we get:

1 = A

Therefore, the value of A is 1.

Writing the equation of the parabola

The equation of the parabola is:

y = (x - 2)² - 2

To know more about parabola:

https://brainly.com/question/11911877

#SPJ2

What type of health screening would this patient most likely receive?
Sue is a 45-year-old woman with a family history of breast cancer. Her healthcare professional will most likely recommend that she receive a .

Answers

Answer:

A mammogram is what she would receive

Step-by-step explanation:

y = RootIndex 3 StartRoot x EndRoot. y = negative (0.4) RootIndex 3 StartRoot x minus 2 EndRoot
Which of the following describes the graph of the transformed function compared with the parent function? Select all that apply.

Answers

Answer:

- Reflected over the x-axis  

- Compressed by a factor of 0.4.

- Translated 2 units left

Step-by-step explanation:

Given

[tex]y = \sqrt[3]{x}[/tex]

[tex]y' = -(0.4)\sqrt[3]{x-2}[/tex]

Required

The transformation from y to y'

First, y is reflected over the x-axis.

The transformation rule is:

[tex](x,y) \to (x,-y)[/tex]

So, we have:

[tex]y = \sqrt[3]{x}[/tex] becomes

[tex]y' = -\sqrt[3]{x}[/tex]

Next, it was compressed by a scale factor of 0.4

The rule is:

[tex]y' = k * y[/tex]

Where k is the scale factor (i.e. k = 0.4)

So, we have:

[tex]y' = 0.4 * -\sqrt[3]{x}[/tex]

[tex]y' = -(0.4)\sqrt[3]{x}[/tex]

Lastly, the function is translated 2 units left;

The rule is:

[tex](x,y) \to (x-2,y)[/tex]

So, we have:

[tex]y' = -(0.4)\sqrt[3]{x - 2}[/tex]

Answers:

-reflected over the x-axis

-translated 2 units right

-compressed by a factor of 0.4

Nathan is 1.55 meters tall. At 1 p.m., he measures the length of a tree's shadow to be 38.15 meters. He stands 32.9 meters away from the tree, so that the tip of his shadow meets the tip of the tree's shadow. Find the height of the tree to the nearest hundredth of a meter .

Answers

Answer:

11.26 m

Step-by-step explanation:

The height of the tree is about 11.25 meters.

What are similar triangles?

When the respective sides are proportional and the corresponding angles are congruent, two triangles are said to be similar.

Given that, the height of the person is 1.55 meters, the length of the tree's shadow is 38.15 meters, and the distance between the person and the tree is 32.9 meters.

Let the height of the tree be x.

Note that the scenario makes two similar triangles.

Since the ratio of the side lengths of similar triangles is proportional, it follows:

(38.15 - 32.9)/1.55 = 38.15/x

5.25/1.55 = 38.15/x

3.39 = 38.15/x

x = 38.15/3.39

x = 11.25

Hence, the height of the tree is about 11.25 meters.

Learn more about similar triangles: https://brainly.com/question/25882965

#SPJ2

Which of the following is the parent function of all absolute value functions?
f(x) = 3x
f(x) = |x|
f(x) = 2|x|
f(x)=x²

Answers

f(x)=|x|

a parent function is the basic function before it has undergone transformations.

What is -2y + -4y. Simplify the answer.

Answers

Step-by-step explanation:

Explanation is in the attachment

hope it is helpful to you

Answer:

[tex]-2y+\left(-4\right)y[/tex][tex]=-2y-4y[/tex][tex]=-6y[/tex]

[tex]-----------[/tex]

hope it helps...

have a great day!!

The cost for 2 adults and 5 children to eat at the local buffet restaurant is \$46.50 . The cost for 1 adult and 1 child is $15. What is the cost of a child's meal ?

Answers

Answer:

c = 5.5

Step-by-step explanation:

2 a + 5 c = 46.50

a + c = 15

a = 15 - c

2(15-c ) +5c = 46.50

30 - 2c +5c = 46.5

3c = 16.5

c = 5.5

Let a be the adults and c be the children.

For the first equation there are 2 adults and 5 children so the equation is:-

2a + 5c = 46.50

And for the second equation there is 1 adult and 1 child. So the equation is:-

a + c = 15

a = 15 - c

You take this formula and substitute it to the above formula (2a + 5c = 46.50)

➡️ 2a + 5c = 46.50

➡️ 2(15 - c) + 5c = 46.50

➡️ 30 -2c + 5c = 46.50

➡️ 5c - 2c = 46.50 - 30
(You collect like terms)

➡️ 3c = 16.5

➡️ c = 16.5/3

➡️ c = 5.5

Greg buys 60 garden plants at a cost price of $2.00 each to sell in his shop. He sells 25 of them at the profit of 75% and 18 of them at the profit of 35%. He sells the rest of the plants for 4/5 of the cost price calculate the profit or loss he makes from selling 60 plants stating if it is a profit or loss

Answers

Answer:

$43.30 profit

Step-by-step explanation:

Total cost of plant:

60*2 = 120

Greg makes total of:

25*(2 + 0.75*2) + 18*(2 + 0.35*2) + (60 - 25 - 18)*2*4/5 = 163.3

Since Greg mare than cost, he has a profit and the amount is:

163.3 - 120 = 43.3

What’s the equation of the line that passes through the point (-4,4) and has a slope of 3/4

Answers

Answer:

y-y1=m(x-x1)

y-4=3/4(x+4)

y=3/4x+7

the area of a rectangular bathroom mirror is 20 square feet. it is 2 feet tall. how wide is it?

Answers

Area = length x width

Fill in the given information

20 = 2 x width

Solve for width by dividing both sides by 2

Width =20/2

Width = 10 feet

Answer:

It is 10 feet wide

Step-by-step explanation:

The area of a rectangle is:

A = Length x width

So if we have the area, finding the width means diving the area by the given length.

In this case, the area of the rectangle is 20 square feet and the length is 2 feet:

20/2 = 10

Therefore the missing width is 10 feet

jordan wants to save to buy a car and decides to open a banking account that is offering an interest rate of 4.5% compounded annually how much will jordan have in the account after 5 years it he deposits $7,000 today?

Answers

Answer:

8,723.27$

Step-by-step explanation:

help and explain//////////////////////////////////////////////

Answers

Answer:

(f + g)(x) = 5x - 3

Step-by-step explanation:

(f + g)(x)

= f(x) + g(x)

= 2x + 4 + 3x - 7 ← collect like terms

= 5x - 3

Answer:

5x - 3

Step-by-step explanation:

(f + g)(x) = f(x) + g(x)

= 2x + 4 + ( 3x - 7 )

= 5x - 3

It took Sarah 4 days to write a paper, she wrote 12 pages on day 1, 15 pages on day 2 and 9 pages on day 3. If she wrote 12 pages per day , how many pages did she write on the fourth day ?

Answers

Answer:

Option A

Step-by-step explanation:

Sarah took 4 days to write a paper.

She wrote 12 pages per day, so total number of pages she wrote in 4 days = 12 × 4

= 48 pages

On day 1, she wrote number of pages = 12

On day 2, she wrote number of pages = 15

On day 3, she wrote number of pages = 9

On day 4, she wrote number of pages = P

She wrote total number pages in 4 days = 12 + 15 + 9 + P

                                                                    = 36 + P

Therefore, P + 36 = 48

P = 48 - 36

P = 12

She wrote 12 pages on day 4.

Option A is the answer.

The formula for the circumference of a circle is R = c/2(pi)

Find the radius of a circle that has a circumference of 16(pi)

A) r = 4
B) r = 8
C) r = 12
D) r = 16

Answers

The answer is B) r= 8

I NEED THE ANSWER ASAP PLZ ;C

Answers

The top right quadrant is quadrant 1.

The top left quadrant is quadrant 2.

The bottom left quadrant is quadrant 3.

The bottom right quadrant is quadrant 4.

Therefore, point D is in the fourth quadrant.

solve for x. Round to the nearest tenth, if necessary.​

Answers

Answer:

7.1

Step-by-step explanation:

We used SOHCAHTOA because it's a right angle triangle

So because we have an angle with an adjacent of 6.3 and hypotenuse of x

We will use

Cos=adjacent /hypotenuse

7.1 like the last answer if not you can round that

find the slope of the line passing through the points (-2,5) and (3/2,2)

Answers

Answer:

slope = - [tex]\frac{6}{7}[/tex]

Step-by-step explanation:

Calculate the slope m using the slope formula

m = [tex]\frac{y_{2}-y_{1} }{x_{2}-x_{1} }[/tex]

with (x₁, y₁ ) = (- 2, 5) and (x₂, y₂ ) = ([tex]\frac{3}{2}[/tex], 2)

m = [tex]\frac{2-5}{\frac{3}{2}-(-2) }[/tex]

   = [tex]\frac{-3}{\frac{3}{2}+2 }[/tex]

   = [tex]\frac{-3}{\frac{7}{2} }[/tex]

  = - 3  × [tex]\frac{2}{7}[/tex]

  = - [tex]\frac{6}{7}[/tex]

Explain how the quotient of powers was used to simplify this expression

Answers

Answer:

See explanation

Step-by-step explanation:

Given:

5⁴/25 = 5²

25 = 5²

Then,

5⁴ / 25

= 5⁴/5²

Note:

• multiplication sign means addition in indices

• Division sign means subtraction in indices

Both numerator and denominator have the same base, so you'll pick one of the bases and subtract the powers

So,

5⁴/5²

= 5^(4 - 2)

= 5^(2)

= 5²

Therefore,

5⁴ / 25 = 5²

2 1/4 x 3 1/5 brainliest

Answers

Answer:

36/5

Step-by-step explanation:

9/4×16/5

144/20

36/5

hope this is helpful

Answer:

[tex]7\frac{1}{5}[/tex]

Step-by-step explanation:

1. start by turning the fractions improper fractions:

[tex]2\frac{1}{4} =\frac{9}{4}[/tex]

[tex]3\frac{1}{5} =\frac{16}{5}[/tex]

2. then multiply them together:

[tex]\frac{9}{4}[/tex] x [tex]\frac{16}{5}[/tex] = [tex]\frac{144}{20}[/tex]

3. then simplify the fraction:

[tex]\frac{144}{20}[/tex][tex]=\frac{36}{5}[/tex]

4. turn it into a proper fraction:

[tex]\frac{36}{5} =7\frac{1}{5}[/tex]

If xy = 6 and x = 2, then x + y =​

Answers

Answer:

5

Step-by-step explanation:

xy= 6

x= 2

x + y = ?

Plug in x=2 in the first equation:

2y=6

y=3

then plug x=2 and y=3 into the one you're solving for:

2+3=5

Hope that helps!

The product of integers a,b,c and d is 546 and if 1<a<b<c<d, what is the value of b+c?​

Answers

Hello,

546=2*3*7*13

a=2, b=3,c=7,d=13

b+c=3+7=10

Select the expression that represents the following statement: add 24 to the quotient of 16 and 8.

Answers

Answer:

16/8 + 24

Step-by-step explanation:

On the unit circle, which of the following angles has the terminal point
coordinates.
A. 45
B. 135
C. 225
D. 315

Answers

Answer: C. 225

Step-by-step explanation:

HELP SOMEONE PLEASE!!!!!!!!!!!!!!!!!!!

Answers

Answer:

3/10

Step-by-step explanation:

The question asks for the probability of finding 4 or more, so first, you must add up all of the teenagers that have 4 or more shoes. Both the 4 and 5 categories fit, so add 20 and 10. Then, find the total number of people in the sample size. Overall, 100 teenagers are represented. Finally, put the number of people that have 4 or more shoes, 30, over the sample size, 100. This equals 30/100 which should be simplified to 3/10.

Sketch the graph of each of the following quadratic functions: (a) f(x) = -2x² + 7x + 4 for -1 ≤ x ≤ 5.
Help me with this ques pleasee,i'll mark u as the brainliest!!​

Answers

Answer:

Please find attached the graph of the function created with MS Excel showing the relevant points required to draw an approximate graph of the function on a graph paper

Step-by-step explanation:

The given quadratic function is f(x) = -2·x² + 7·x + 4

The range of the input (x) values = -1 ≤ x ≤ 5

The coefficient of the quadratic is negative -2, the graph is n shape

The intercept form of the function is given as follows;

-2·x² + 7·x + 4 = -1 × (2·x² - 7·x - 4)

-1 × (2·x² - 7·x - 4) = -1 × (2·x² + x - 8·x - 4)

-1 × (2·x² + x - 8·x - 4) = -1 × (x · (2·x + 1) - 4·(2·x + 1))

∴ -1 × (x · (2·x + 1) - 4·(2·x + 1)) = -1 × (2·x + 1)·(x - 4)

∴ f(x) = -2·x² + 7·x + 4 = -1 × (2·x + 1)·(x - 4)

At the x-intercepts, (2·x + 1) = 0 or (x - 4) = 0, which gives;

x = -1/2 or x = 4

Therefore, the x-intercepts are (-1/2, 0), (4, 0)

The equation in vertex form is given as follows;

f(x) = -2·x² + 7·x + 4 = -2·(x² - 7·x/2 + 2)

By applying completing the squares method, to x² - 7·x/2 - 2, we get;

Where x² - 7·x/2 - 2

x² - 7·x/2 = 2

x² - 7·x/2 + (-7/4)² = 2 + (-7/4)² = 81/15

(x - 7/4)² = 81/16

∴ (x - 7/4)² - 81/16 = 0 = x² - 7·x/2 - 2

∴ x² - 7·x/2 - 2 = (x - 7/4)² - 81/16

-2·(x² - 7·x/2 + 2) = -2·((x - 7/4)² - 81/16) = -2·(x - 7/4)² + 81/8

The vertex = (7/4, 81/8)

When x = 0, we get;

f(0) = -2 × 0² + 7 × 0 + 4 = 4

The y-intercept = (0, 4)

The sketch of the function should pass through the x-intercepts (-1/2, 0), (4, 0), the y-intercept (0, 4), and the y-intercept (0, 4), and the vertex, (7/4, 81/8) on a graph sheet

Please find attached a drawing of the function of the function created with MS Excel

According to the number line, what is the distance between points A and B?

0 6 units
7 units
O 12 units
O 14 units

Answers

Answer:

14 units

Step-by-step explanation:

A = - 2, B = 12

Therefore,

d(A, B) = 12 - (-2) = 12 + 2 = 14 units

solve in attachment .​

Answers

Answer:

2 ( Option A )

Step-by-step explanation:

The given integral to us is ,

[tex]\longrightarrow \displaystyle \int_0^1 5x \sqrt{x}\ dx [/tex]

Here 5 is a constant so it can come out . So that,

[tex]\longrightarrow \displaystyle I = 5 \int_0^1 x \sqrt{x}\ dx [/tex]

Now we can write √x as ,

[tex]\longrightarrow I = \displaystyle 5 \int_0^1 x . x^{\frac{1}{2}} \ dx [/tex]

Simplify ,

[tex]\longrightarrow I = 5 \displaystyle \int_0^1 x^{\frac{3}{2}}\ dx [/tex]

By Power rule , the integral of x^3/2 wrt x is , 2/5x^5/2 . Therefore ,

[tex]\longrightarrow I = 5 \bigg( \dfrac{2}{5} x^{\frac{5}{2}} \bigg] ^1_0 \bigg) [/tex]

On simplifying we will get ,

[tex]\longrightarrow \underline{\underline{ I = 2 }}[/tex]

Step-by-step explanation:

[tex]thank \: you[/tex]

Cayden has several screws on a scale, and the scale reads 80.955 cayden add 1 more screw and the scale reads 84.81

Answers

Answer:

-3.855

Step-by-step explanation:

Not sure if you need the weight of each screw! but you just subtract 80.955-84.81

Answer:

1

Step-by-step explanation:

itnisbbsndnfnfnfnnfnfnfncjcjccjcj

Help me?
185/100 + 50% + 4%=?

Answers

Answer:

2.39

Step-by-step explanation:

Convert everything to a decimal.  

To do that with a fraction divide the numerator by the denominator.  

185 divided by 100 = 1.85

185/100 = 1.85

To convert a percent to a decimal divide the number by 100.

50 divided by 100 = 0.5

50/100 = 0.5

4 divided by 100 = 0.04

4/100 = 0.04

Now add:

1.85 + 0.5 + 0.04 = 2.39

Other Questions
The last dividend paid by Wilden Corporation was $1.55. The dividend growth rate is expected to be constant at 1.5% for 2 years, after which dividends are expected to grow at a rate of 6.0% forever. The firm's required return (rs) is 12.0%. What is the best estimate of the current stock price? Derek decides to buy a new car. The dealership offers him a choice of paying $600.00 per month for 5 years (with the first payment due next month) or paying some amount today. He can borrow money from his bank to buy the car. The bank requires a 5.00% interest rate. What is the most that he would be willing to pay today rather than making the payments Question:which is a y-intercept of the graphed function?Answers:A. (-9,0)B. (-3,0)C. (0,-9) D. (0,-3) Which is a possible consequence of decreasing air quality resulting from global warming? drying out of fertile lands flooding of low-lying coastal areas melting of glaciers and polar ice caps decrease in population due to health threats The correct answer is decrease in population due to health threats. If you are camping in a rural setting and look up at the night sky, it can appear as if ________. You are an off-duty officer who has just completed a three-mile run. You decide to cool off and rest under a big oak tree. You sit down under the tree and see a brown wallet close by. Curiosity gets the best of you, and you open the waller. Interestinglyenough, you find credit cards, identification, and $300 in cash. Nobody else is around. What would you do? What is the surface area of the composite solid?A. 119 m2B. 146 m2C. 162 m2D. 174 m2 Can someone solve this for me and a couple more questions ? Which functions have a range of {y e R | - Space weather is dominated by a stream of charged particles emanating from the sun's magnetic field, known as ______________. Two alternate plans are available for increasing the capacity of existing water transmission lines between an unlimited source and a reservoir. The unlimited source is at a higher elevation then the reservoir. Plan A calls for the construction of a parallel pipeline and flow by gravity. Plan B specifies construction of a booster pumping station. Estimated cost for the two plans are as follows: Hint: Use Present Worth- (do not guess show all your work) i=10% Plan A : Cost $700,000, Life 40 Years, Annual Operation and Repair $1,000/Year Plan B: Cost $200,000, Life 40 Years Structure and 20 years equipment, Equipment replacement at the end of 20 years $75,000, Annual Operation and Repairs 52,000/yeara. Plan A $709,779.00 b. Plan A $740,000 c. Plan B $710,165.50 d. Plan B $326,000 write a letter to your district chief executive telling him/ her three problem facing my locality Rank each of the following gases in order of increasing urms assuming equivalent amounts and all gases are at the same temperature and pressure where 1 has the lowest urms and 4 has the highest urms.a. Gas 1 : H2Sb. Gas: Hec. Gas 3: NF3d. Gas 4: H2O A 70-year-old male presents with back pain, fever, and weight loss. He reports that he had a recent respiratory infection from which he thought he recovered. Tests revealed increased white blood cell count, and a diagnosis of endogenous osteomyelitis was made. The primary organism causing this condition is Find the area of circle Q in terms of x Molecules that do not contain carbon are called inorganic. true false Chang knows one side of a triangle is 13 cm. Which set of two sides is possible for the lengths of the other twosides of this triangle?5 cm and 8 cm6 cm and 7 cm7 cm and 2 cm8 cm and 9 cm If velocity is constant and, in addition, the factors of production and the production function determine real GDP, then: Group of answer choices nominal GDP is fixed. the price level is proportional to the money supply. real GDP is proportional to the money supply. the price level is fixed. A cules grupos est dirigido este artculo ?A. A los miembros del Banco Mundial y a donantes potenciales B. A los msicos y los estadounidenses C. A la gente interesada en la lucha contra la pobreza.D. A los nios pobres y sus padres. ask a question and have your peers answer: heres my question which rapper is better NFEMINEMXXXtentacionJuice Wrld